0 Daumen
83 Aufrufe

Aufgabe:

Sei F eine Verteilungsfunktion einer reellen Zufallsvariablen X und F-1 die inverse Verteilungsfunktion, wobei F-1(x) := inf(y ∈ ℝ: F(y) >= x). Zeigen Sie, dass F-1 linksstetig ist   


Problem/Ansatz:

Hallo Leute! Ich habe hier bereits einen Beweis für den oben stehenden Satz, aber ich komme bei dem rot markierten Teil nicht weiter. Ich verstehe den Teil einfach nicht. Vielleicht gibt es unter uns eine hellere Birne als mich, die mir weiterhelfen kann? Beste Grüße!


Screenshot 2024-03-18 222544.png

Avatar von

1 Antwort

0 Daumen
 
Beste Antwort

Vorher wurde \( F(z) \geq x_n \) gezeigt. Also ist \( F(z) \) eine obere Schranke für die Folge. Daher ist auch \( F(z) \geq x \) (1).

Im Schritt darunter wird dann die Annahme \( F^{-1}(x) > z \) benutzt und zum Widerspruch geführt, denn im letzten Schritt wird die Monotonie von \( F^{-1} \) auf (1) angewendet.

Avatar von 11 k

ach man, wenn man den Wald vor lauter Bäumen nicht sieht. Vielen vielen Dank! :)

Ein anderes Problem?

Stell deine Frage

Willkommen bei der Mathelounge! Stell deine Frage einfach und kostenlos

x
Made by a lovely community